LSAT and Law School Admissions Forum

Get expert LSAT preparation and law school admissions advice from PowerScore Test Preparation.

 Administrator
PowerScore Staff
  • PowerScore Staff
  • Posts: 8916
  • Joined: Feb 02, 2011
|
#38552
Complete Question Explanation
(The complete setup for this game can be found here: lsat/viewtopic.php?t=15133)

The correct answer choice is (C)

If M is published in the fall, then P and N must be published in the spring (first and fourth rules, respectively). This eliminates answer choices (B), (D), and (E). Since K and N must be in the same group as each other, it is clear that K must also be published in the spring, eliminating answer choice (A). Thus, answer choice (C) is proven correct.

Note that if K is published in the spring, O can be published in either season. The third rule only tells us what must be true about O if K is published in the fall (or, by the contrapositive, what must be true about K if O is published in the spring). It is entirely possible, however, that K is published in the spring and O—in the fall.

If you were using templates here, it is clear that only Template 2 can provide a suitable solution to the local condition of this question. According to that template, if M is published in the fall, then P, K, and N must all be published in the spring. L and O are the only random variables here, making answer choice (C) a sure bet.

Get the most out of your LSAT Prep Plus subscription.

Analyze and track your performance with our Testing and Analytics Package.